A 30KW motor running at 1450 rpm draws 56A, how would you
determine if this is correct or not?

Answer Posted / simon

30 KW=440(V)*68.1818(Amp).

Is This Answer Correct ?    0 Yes 0 No



Post New Answer       View All Answers


Please Help Members By Posting Answers For Below Questions

why we setting 95 deg and 105 deg in wti and oti of distibution transformer ? give the reson for that ?

2585


What is process of anneling transformer core

1462


Define reserve-generating capacity?

655


What are the protection required for Transformer, motor, Line, Generator, capacitor, that all i have to refer which standard?(Like Transformer detail IS 2026)

1467


compare between starting charactristics of synchronous motor with those of 3Q Squirrel cage induction motor

1273






describe the phenomena of vacuum interruption in C.B

1250


Explain the terms real power, apparent power and reactive power for ac circuits and also the units used.

633


How to calculate the Efficiency of Chiller

2854


we have require a Motor (1500kW, 6.6kV, 50Hz) which shall operate at 1500rpm during 30min and after it shall run on 500rpm during 15min. The same cycle shall be repetitive during the whole working days. Which method is best and economic to achieve such type of task. VFD, VVFD, Pole changing motor etc.

1490


WHAT HAPPENED DC SERIES MOTOR FIELD WINDING OPEN. CAN POSSIBLE.

3207


in a solar power plant,we are going to step up the voltage 350V to 33kv, and to 132kv GSS. i want to ask that if it is possible that 315v/33kv is not possible because we also use 315v/11kv. or it should be 415v/33kv? is it effects generation or losses?  the second question is-generation is depends on LV winding? or LV winding design for generation voltage?

1295


Does magnetomonopoles exists? one maxwell equation strongly suggests no, but how about their existance in plasma?

1475


List advantges of active filter over passive filter?

632


what about electrical pannel.? What is meant by plc pannel.?

1871


how to minimize earth resistance upto 2Ω, if the resistance is 6 Ω ?

1452